a8l367
Thanks Received: 1
Jackie Chiles
Jackie Chiles
 
Posts: 44
Joined: July 22nd, 2017
 
 
 

Re: Q21 - Newspaper editor: Law enforcement experts

by a8l367 Wed Mar 21, 2018 8:22 am

ManhattanPrepLSAT1 Wrote:By definition, assumptions are unstated. So answer choice (C) is not an assumption, but rather a premise. So no, you cannot say that answer choice (C) is an assumption.

Hope that helps!

Passage:
the laws are impossible to enforce (as i understant the means current (or past) laws, not every law possible ever)

C:
No legal prohibitions against gambling are enforceable. (as I understand it's about all possible legal prohibitions, including those, not created yet)

Please clarify
 
ZhaoX24
Thanks Received: 0
Vinny Gambini
Vinny Gambini
 
Posts: 1
Joined: May 31st, 2018
 
 
 

Re: Q21 - Newspaper editor: Law enforcement experts

by ZhaoX24 Mon Jun 04, 2018 12:02 pm

ohthatpatrick Wrote:Consider any claim of the form "No A's are B".

No lions can speak English.


Shall we represent that as
If you're not a lion, then you can speak English
or
If you are a lion, then you can't speak English

The 2nd one is what we want. "No A's are B" = If A, then not-B

So translate (A) accordingly:
If you're an effective law, then you're not-unenforceable.
i.e. If you're an effective law, then you're enforceable.

The contrapositive of that rule would be,
If you're not-enforceable, then you're not-effective law.

According to our evidence, "when a law fails to be effective it shouldn't be a law."
If we add the rule from (A), we know that
If a law is not enforceable, it's not effective, and thus it shouldn't be a law.

We were told that gambling laws are not enforceable, so this chained together rule lets us derive that gambling laws shouldn't be laws.


Hi, My question maybe very simple but it really make me confused. I read the 'logic reasoning guide' and all the message above, I agree with the right answer. But still I am not sure how can we translate '(A) No effective law is unenforceable' into 'if a law is effective, then it must be enforceable'.

In other words, we have 'No effective law is unenforceable', so I say 'if law is not effective, then it is unenforceable'. And then I write it in conditional form like ' no effective -> unenforceable'(this can not become effective law-> enforceable law). Then I can make a valid inference by saying that 'enforceable -> effective'. Therefore, (A) doesn't seems to me to be a right answer...

Could you please help me to figure out my fault ?

Thank you
 
MichaelaA384
Thanks Received: 0
Vinny Gambini
Vinny Gambini
 
Posts: 1
Joined: December 05th, 2018
 
 
 

Re: Q21 - Newspaper editor: Law enforcement experts

by MichaelaA384 Wed Dec 05, 2018 4:56 pm

I've read through all the posts/replies regarding this LR question and am still confused :?:

The gap is plain. That we need to make a connection between enforcement and effectiveness in order to justify the conclusion that there should be no legal prohibition against gambling.

I've narrowed my answers down to A and B, however I'm very confused as to how those answer choices differ...

A) "no effective law is unenforceable."
Would this not be ~effective law -> ~enforceable ?
B) "all enforceable laws are effective."
Would this not be enforceable law -> effective, the contrapositive of which is ~effective ->~enforceable and therefore the same as A?

I'm assuming here that I must be diagramming something incorrectly but I am unsure where I went wrong.. Thanks for the help!!
 
BensonC202
Thanks Received: 0
Vinny Gambini
Vinny Gambini
 
Posts: 19
Joined: April 08th, 2019
 
 
 

Re: Q21 - Newspaper editor: Law enforcement experts

by BensonC202 Wed Jul 15, 2020 5:45 pm

giladedelman Wrote:Very good post!

So, your reasoning is 99% accurate. Let's walk through this question one more time.

Like you said, we have two premises that lead to our conclusion, so the core looks like this:

anti-gambling laws unenforceable
if a law is ineffective, shouldn't be a law --> anti-gambling laws shouldn't exist

Now, again as you pointed out, the gap here is pretty clear. The only thing we know about anti-gambling laws is that they're unenforceable. The only way we know whether a law should exist is whether it's effective. So we can expect the assumption to connect "unenforceable" to "ineffective" -- something like, if it's unenforceable, it's not effective. (The only thing I didn't like in your explanation is where you said "the author needs to assume that gambling laws are not effective because they are impossible to enforce." We don't need a causal connection. But I'm nitpicking.)

(A) is correct because if no effective law is unenforceable, no unenforceable law is effective, so the anti-gambling laws are ineffective and we should get rid of them.

(B) is the negated version of what we want. It doesn't tell us anything useful about unenforceable laws.

(C) is just a restatement of the premise that anti-gambling laws are unenforceable.

(D) and (E) are both out of scope because whether citizens agree doesn't affect our core whatsoever.

Now, notice that this question asks, "Which one of the following, if assumed, allows the argument’s conclusion to be properly drawn?" That means we're looking for a sufficient assumption, one that gets us all the way to the conclusion.

So, to answer your question, yes: if answer (A) said "anti-gambling laws are ineffective," that would be correct, because it would be enough to get us to the conclusion. But you were also right to prepare to find an assumption that linked the two premises, since that's the tendency on assumption family questions.

Does that satisfy you? Good explanation.



In terms of options C, may we also interpret option C as it tends to equate the characteristic of the subgroup of law, legal prohibition against gambling, to the overall group of law ? Since the second premises clearly stated that if a law is not effective, there should be no law, and the conclusion extrapolates that there should not be legal prohibition against gambling.

Option C = Any legal prohibition against gambling is not enforceable; however, it does not mean that the law other than the legal prohibition against gambling is also " not enforceable " either, and if its true, the subjects be discussed within both the 1 & 2 premises and option 3 is not the same, which makes it impossible to connect the sufficient assumption as correct option A does.

P1 = Regardless of the efforts, " All " laws are impossible to enforce.

P2= Any law fail to be effective, it should not be a law.

C = Legal prohibition against gambling ( law of one subgroups of law ) should not be existed.

Apparently, the correct answer is A, Effective law ---> enforceable, and, as my humble 2 cents to share, we may interpret option C in a different way other than regarding it simply as the premise booster.

Please critic and correct me if I am wrong. Deeply appreciate it.